Which indicates the first law of thermodynamics ​

Which Indicates The First Law Of Thermodynamics

Answers

Answer 1

Answer:

(d)

Explanation:

because dU = Q -W so ,that the option d(D) is correct


Related Questions

When the insulation resistance between a motor winding and the motor frame is tested, the value obtained is 1.0 megohm (106 Ω). How much current passes through the insulation of the motor if the test voltage is 1000 V?​

Answers

Answer:

0.001 A

Explanation:

Applying,

V = IR.............. Equation 1

Where V = Voltage of the motor, I = current, R = resistance

make I the subject of the equation

I = V/R.............. Equation 2

From the question,

Given: V = 1000 V, R = 1 MΩ = 10⁶ Ω

Substitute these values into equation 2

I = 1000/10⁶

I = 10⁻³ A

I = 0.001 A

Explain the following defects of a simple electric cell:

a.Polarization,

ß. Local action.​

Answers

Answer:

Explanation:

The two major defects of simple electric cells causes current supplied to be for short time. These defects are: polarization and local action.

a. Polarization: This is a defect caused by an accumulation of hydrogen bubbles at the positive electrode of the cell. It can be prevented by the use of vent, using a hydrogen absorbing material or the use of a depolarizer.

b. Local Action: This is the gradual wearing away of the electrode due to impurities in the zinc plate. It can be controlled by the amalgamation of the zinc plate before it is used.

A 10.0kg of desk initial is pushed along a frictionless surface by a constant horizontal of force magnitude 12N Find the speed of the desk after it has moved through a horizontal distance of 5.0m ​

Answers

kylydljty many true dvx*&;'*+$_5+

i.Name two commonly used thermometric liquids.


ii.State two advantages each of the thermometric liquids mentioned above​

Answers

Answer:

mercury and alcohol

ii) used to test temperatures

Mercury and AlcoholMercury:

i) It is a good conductor of heat and therefore the whole liquid reaches the temperature of the surroundings quickly.

ii) It does not wet (cling to the sides of) the tube.

Alcohol:

i) Alcohol has greater value of temperature coefficient of expansion than mercury.

ii) it's freezing point is below –100°C.

A grade 12 Physics student shoots a basketball
from the ground at a hoop which is 2.0 m above
her release. The shot was at a velocity of 10 m/s
and at an angle of 80° to the ground.
a. Determine the vertical velocity of the ball
when it is at the level of the net. You
should get two answers.
Please show ALL steps

Answers

Answer:

7.84 m/s

Explanation:

Height, h = 2 m

Initial velocity, u = 10 m/s

Angle, A = 80°

(a) Let the time taken to go to the net is t.

Use second equation of motion

[tex]h = u t + 0.5 at^2\\\\- 2 = - 10 sin 80 t - 4.9 t^2\\\\4.9 t^2 + 9.8 t - 2 = 0 \\\\t= \frac{- 9.8\pm\sqrt{9.8^2 + 4\times 4.9\times 2}}{9.8}\\\\t = \frac{- 9.8 \pm 11.6}{9.8}\\\\t = - 2.2 s , 0.2 s[/tex]

Time cannot be negative.

So, t = 0.2 s

The vertical velocity at t = 0.2 s is

v = u + at

v = 10 sin 80 - 9.8 x0.2

v = 9.8 - 1.96 = 7.84 m/s

A bullet with mass 5.35 g is fired horizontally into a 2.174-kg block attached to a horizontal spring. The spring has a constant 6.17 102 N/m and reaches a maximum compression of 6.34 cm.
(a) Find the initial speed of the bullet-block system.
(b) Find the speed of the bullet.

Answers

Answer:

a)[tex]V=1.067\: m/s[/tex]

b)[tex]v=434.65\: m/s [/tex]  

Explanation:

a)

Using the conservation of energy between the moment when the bullet hit the block and the maximum compression of the spring.

[tex]\frac{1}{2}MV^{2}=\frac{1}{2}k\Delta x^{2}[/tex]

Where:

M is the bullet-block mass (0.00535 kg + 2.174 kg = 2.17935 kg)V is the speed of the systemk is the spring constant (6.17*10² N/m)Δx is the compression of the spring (0.0634 m)

Then, let's find the initial speed of the bullet-block system.

[tex]V^{2}=\frac{k\Delta x^{2}}{M}[/tex]

[tex]V=\sqrt{\frac{6.17*10^{2}*0.0634^{2}}{2.17935}}[/tex]

[tex]V=1.067\: m/s[/tex]

b)

Using the conservation of momentum we can find the velocity of the bullet.

[tex]mv=MV[/tex]

[tex]v=\frac{MV}{m}[/tex]

[tex]v=\frac{2.17935*1.067}{0.00535}[/tex]

[tex]v=434.65\: m/s [/tex]  

I hope it helps you!

             

 

7. If a load of 300N is pulled along the inclined plane shown in the figure, answer the following. B 200 N 0.5m 2m 300 N А i. Calculate the VR and MA of the inclined plane. Calculate the input work and output work. ii.what efficiency of inclined plane?iv.what should be the length of inclined plane if same load has to be pulled with a50N effort for the same efficiency as above​

Answers

Explanation:

700n I think friend .. if worng

SCALCET8 3.9.018.MI. A spotlight on the ground shines on a wall 12 m away. If a man 2 m tall walks from the spotlight toward the building at a speed of 1.7 m/s, how fast is the length of his shadow on the building decreasing when he is 4 m from the building

Answers

Answer:

The length of his shadow is decreasing at a rate of 1.13 m/s

Explanation:

The ray of light hitting the ground forms a right angled triangle of height H, which is the height of the building and width, D which is the distance of the tip of the shadow from the building.

Also, the height of the man, h which is parallel to H forms a right-angled triangle of width, L which is the length of the shadow.

By similar triangles,

H/D = h/L

L = hD/H

Also, when the man is 4 m from the building, the length of his shadow is L = D - 4

So, D - 4 = hD/H

H(D - 4) = hD

H = hD/(D - 4)

Since h = 2 m and D = 12 m,

H = 2 m × 12 m/(12 m - 4 m)

H = 24 m²/8 m

H = 3 m

Since L = hD/H

and h and H are constant, differentiating L with respect to time, we have

dL/dt = d(hD/H)/dt

dL/dt = h(dD/dt)/H

Now dD/dt = velocity(speed) of man = -1.7 m/s ( negative since he is moving towards the building in the negative x - direction)

Since h = 2 m and H = 3 m,

dL/dt = h(dD/dt)/H

dL/dt = 2 m(-1.7 m/s)/3 m

dL/dt = -3.4/3 m/s

dL/dt = -1.13 m/s

So, the length of his shadow is decreasing at a rate of 1.13 m/s

now suppose that we have attached not just two springs in series, but N springs. Write an equation that expresses the effective spring constant of the combination using the spring constant of the original spring k and the number of springs N

Answers

Answer:

 [tex]k_{eq} = \frac{k}{N}[/tex]

Explanation:

For this exercise let's use hooke's law

         F = - k x

where x is the displacement from the equilibrium position.

        x = [tex]- \frac{F}{k}[/tex]

if we have several springs in series, the total displacement is the sum of the displacement for each spring, F the external force applied to the springs

       x_ {total} = ∑ x_i

we substitute

       x_ {total} =  ∑ -F / ki

       F / k_ {eq} =  -F  [tex]\sum \frac{1}{k_i}[/tex]

      [tex]\frac{1}{k_{eq}} = \frac{1}{k_i}[/tex] 1 / k_ {eq} =  ∑ 1 / k_i

if all the springs are the same

     k_i = k

     [tex]\frac{1}{k_{eq}} = \frac{1}{k} \sum 1 \\[/tex]

     [tex]\frac{1}{k_{eq} } = \frac{N}{k}[/tex]

     [tex]k_{eq} = \frac{k}{N}[/tex]

1. What is the total distance the car moves until it stops?
a. 250 m
b. 450 m
c. 300 m
d. 600 m.​

Answers

B
Just took the quiz bro it was easy

Alice and Tom dive from an overhang into the lake below. Tom simply drops straight down from the edge, but Alice takes a running start and jumps with an initial horizontal velocity of 25 m/s. Neither person experiences any significant air resistance. Compare the time it takes each of them to reach the lake below.

a. Alice reaches the surface of the lake first
b. Tom reaches the surface of the lake first
c. Alice and Tom will reach the surface of the lake at the same time.

Answers

It would be a Alexis reached the surface of the lake first

The relation of mass m, angular velocity o and radius of the circular path r of an object with the centripetal force is-
a. F = m²wr
b. F = mwr²
c. F = mw²r
d. F = mwr. ​

Answers

Answer:

Correct option not indicated

Explanation:

There are few mistakes in the question. The angular velocity ought to have been denoted with "ω" and not "o" (as also suggested in the options).

The formula to calculate a centripetal force (F) is

F = mv²/r

Where m is mass, v is velocity and r is radius

where

While the formula to calculate a centrifugal force (F) is

F = mω²r

where m is mass, ω is angular velocity and r is radius of the circular path.

From the above, it can be denoted that the relationship been referred to in the question is that of a centrifugal force and not centripetal force, thus the correct option should be C.

NOTE: Centripetal force is the force required to keep an object moving in a circular path/motion and acts inward towards the centre of rotation while centrifugal force is the force felt by an object in circular motion which acts outward away from the centre of rotation.

Distillation is the separation of multiple Choose... components based on their different Choose... . As the mixture is heated and the first component Choose... , its Choose... form travels through the distillation set-up and Choose... into a different container.

Answers

Answer:

Explanation:

Distillation is the separation of multiple LIQUID components based on their different BOILING POINT. As the mixture is heated and the first component SEPARATES, its PURE form travels through the distillation set-up and GOES into a different container

There are two beakers of water on the table. We can compare the average kinetic energy of the water molecules in the two beakers by measuring their

A temperatures.

B volumes.

C densities.

D masses.

Answers

Answer: masses

Explanation:

Trust me

Find the volume of cuboid of side 4cm. Convert it in SI form​

Answers

Answer:

0.000064 cubic meters.

Explanation:

Given the following data;

Length of side = 4 centimeters

Conversion:

100 centimeters = 1 meters

4 cm = 4/100 = 0.04 meters

To find the volume of cuboid;

Mathematically, the volume of a cuboid is given by the formula;

Volume of cuboid = length * width * height

However, when all the sides are equal the formula is;

Volume of cuboid = L³

Volume of cuboid = 0.04³

Volume of cuboid = 0.000064 cubic meters.

A tank is full of water. Find the work (in J) required to pump the water out of the spout. (Use 9.8 m/s2 for g. Use 1,000 kg/m3 as the density of water. Round your answer to the nearest whole number.)

Answers

6 m in 26,000 26 m in 27

In the late 19th century, great interest was directed toward the study of electrical discharges in gases and the nature of so-called cathode rays. One remarkable series of experiments with cathode rays, conducted by J. J. Thomson around 1897, led to the discovery of the electron.
With the idea that cathode rays were charged particles, Thomson used a cathode-ray tube to measure the ratio of charge to mass, q/m, of these particles, repeating the measurements with different cathode materials and different residual gases in the tube.
Part A
What is the most significant conclusion that Thomson was able to draw from his measurements?
He found a different value of q/m for different cathode materials.
He found the same value of q/m for different cathode materials.
From measurements of q/m he was able to calculate the charge of an electron.
From measurements of q/m he was able to calculate the mass of an electron.
Part B
What is the distance Δy between the two points that you observe? Assume that the plates have length d, and use e and m for the charge and the mass of the electrons, respectively.
Express your answer in terms of e, m, d, v0, L, and E0.
Part C
Now imagine that you place your entire apparatus inside a region of magnetic field of magnitude B0 (Figure 2) . The magnetic field is perpendicular to E⃗ 0 and directed straight into the plane of the figure. You adjust the value of B0 so that no deflection is observed on the screen.
What is the speed v0 of the electrons in this case?
Express your answer in terms of E0 and B0.

Answers

Answer:

a) He found the same value of q/m for different cathode materials.

b)      y = [tex]- \frac{e}{m}\ \frac{E_o v_o^2 }{2d^2}[/tex] ,  c)  v = [tex]\frac{E_o}{B_o}[/tex]

Explanation:

In Thomson's experiments he was able to measure the deflection of the light beam under the effect of the magnetic field and with these results find the e / m relationship, which in all cases is the same, therefore the most important conclusion is that the value e E / m is constant for all materials.

b) In the part of the plates the electrons are accelerated by the electric field,

              F = ma

             - e E = m a

              a = - (e/m)  E₀

               

the distance traveled is          

X axis

          x = v₀ t

the separation of the plates is x = d

          t = vo / d

               

Y axis

          y = v_{oy} t + ½ to t²

          y = ½ a t²

          y = [tex]- \frac{e}{m}\ \frac{E_o v_o^2 }{2d^2}[/tex]

           

c) In this case there is a magnetic field B₀ and the electrons have no deflection

         F = - e E + e v x B

       

if there is no deviation F = 0

         e E = e v B

         v = [tex]\frac{E_o}{B_o}[/tex]

The electric potential ( relative to infinity ) due to a single point charge Q is 400 V at a point that is 0.6 m to the right of Q. The electric potential (relative to infinity) at a point that is 0.90 m to the left of 0 is:_____.
A. + 400 V.
B. -400 V.
C. + 200 V.

Answers

Answer:

The potential at a distance of 0.9 m is 266.67 V.

Explanation:

Charge = Q

Potential is 400 V at a distance 0.6 m .

Let the potential is V at a distance 0.9 m.

Use the formula of potential.

[tex]V = \frac{Kq}{r}\\\\\frac{V}{400}=\frac{0.6}{0.9}\\\\V = 266.67 V[/tex]

Why is the force of attraction between the Earth and ourselves so huge compared to the attraction between two apples?

Answers

Answer:

Answer in explanation

Explanation:

The force of attraction between two bodies is governed by Newton's Law of Gravitation:

[tex]F = \frac{Gm_1m_2}{r^2}[/tex]

where,

G = Universal Gravitational Constant

m₁ = mass of the first body

m₂ = mass of the second body

r = distance between the two bodies

F = Force

Hence, it is clear from the formula that the magnitude of the force is directly proportional to the product of the masses of the objects. So in the case of the earth and ourselves, the mass of the earth is very large in order of 10²⁴ kg. Due to this huge mass, the attraction between the earth and ourselves is so huge as compared to the attraction between two apples. Because the masses of the apple are very small in grams.

two resistors with resistance values 4.5 ohms and 2.3 ohms are connected in series or parallel across a potential difference of 30V to a light bulb find the current flowing through the light bulb in both cases​

Answers

Answer:

Look at work

Explanation:

Series:

I is the same for all resistors so just find the value of Req. In series Req= R1+R2+...+Rn. So here it will be 4.5+2.3=6.8ohms. Ieq=Veq/Req=4.41A. And since current is the same across all resistors the current to the lightbulb is 4.41A.

Parallel:

V is the same for all resistors so start of by finding Req. In parallel, Ieq=I1+I2+...+In. So I1= 30/4.5= 6.67A and I2= 13.04A. Ieq= 6.67+13.04= 19.71A.

A uniformly dense solid disk with a mass of 4 kg and a radius of 4 m is free to rotate around an axis that passes through the center of the disk and perpendicular to the plane of the disk. The rotational kinetic energy of the disk is increasing at 21 J/s. If the disk starts from rest through what angular displacement (in rad) will it have rotated after 3.3 s?

Answers

Answer:

3.44 rad

Explanation:

The rotational kinetic energy change of the disk is given by ΔK = 1/2I(ω² - ω₀²) where I = rotational inertia of solid sphere = MR²/2 where m = mass of solid disk = 4 kg and R = radius of solid disk = 4 m, ω₀ = initial angular speed of disk = 0 rad/s (since it starts from rest) and ω = final angular speed of disk

Since the kinetic energy is increasing at a rate of 21 J/s, the increase in kinetic energy in 3.3 s is  ΔK = 21 J/s × 3.3 s = 69.3 J

So, ΔK = 1/2I(ω² - ω₀²)

Since ω₀ = 0 rad/s

ΔK = 1/2I(ω² - 0)

ΔK = 1/2Iω²

ΔK = 1/2(MR²/2)ω²

ΔK = MR²ω²/4

ω² = (4ΔK/MR²)

ω = √(4ΔK/MR²)

ω = 2√(ΔK/MR²)

Substituting the values of the variables into the equation, we have

ω = 2√(ΔK/MR²)

ω = 2√(69.3 J/( 4 kg × (4 m)²))

ω = 2√(69.3 J/[ 4 kg × 16 m²])

ω = 2√(69.3 J/64 kgm²)

ω = 2√(1.083 J/kgm²)

ω = 2 × 1.041 rad/s

ω = 2.082 rad/s

The angular displacement θ is gotten from

θ = ω₀t + 1/2αt² where ω₀ = initial angular speed = 0 rad/s (since it starts from rest), t = time of rotation = 3.3 s and α = angular acceleration = (ω - ω₀)/t = (2.082 rad/s - 0 rad/s)/3.3 s = 2.082 rad/s ÷ 3.3 s = 0.631 rad/s²

Substituting the values of the variables into the equation, we have

θ = ω₀t + 1/2αt²

θ = 0 rad/s × 3.3 s + 1/2 × 0.631 rad/s² (3.3 s)²

θ = 0 rad + 1/2 × 0.631 rad/s² × 10.89 s²

θ = 1/2 × 6.87159 rad

θ = 3.436 rad

θ ≅ 3.44 rad

A massless, hollow sphere of radius R is entirely filled with a fluid such that its density is p. This same hollow sphere is now compressed so that its radius is R/2, and then it is entirely filled with the same fluid as before. As such, what is the density of the compressed sphere?
a. 8p
b. p/8
c. p/4
d. 4p

Answers

Answer:

a. 8p

Explanation:

We are given that

Radius of hollow sphere , R1=R

Density of hollow sphere=[tex]\rho[/tex]

After compress

Radius of hollow sphere, R2=R/2

We have to find density of the compressed sphere.

We know that

[tex]Density=\frac{mass}{volume}[/tex]

[tex]Mass=Density\times volume=Constant[/tex]

Therefore,[tex]\rho_1 V_1=\rho_2V_2[/tex]

Volume of sphere=[tex]\frac{4}{3}\pi r^3[/tex]

Using the formula

[tex]\rho\times \frac{4}{3}\pi R^3=\rho_2\times \frac{4}{3}\pi (R/2)^3[/tex]

[tex]\rho R^3=\rho_2\times \frac{R^3}{8}[/tex]

[tex]\rho_2=8\rho[/tex]

Hence, the density of  the compressed sphere=[tex]8\rho[/tex]

Option a is correct.

Question 3 of 10
Which statement describes the law of conservation of energy?
A. Air resistance has no effect on the energy of a system.
B. Energy cannot be created or destroyed.
C. The total energy in a system can only increase.
D. Energy cannot change forms.
هما
SUBMIT

Answers

Answer:

B . energy cannot be created or destroyed

A truck moves 70 m east, then moves 120 m west, and finally moves east again a distance of 90 m. If east is chosen as the positive direction, what is the truck's resultant displacement

Answers

Answer:

140m east

Explanation:

If East is positive then lets rephrase the problem into integers

A truck moves +70 m, then moves -120m, and finally moves +90m.

So totally Displacement = +70-120+90= +140m

Since east is positive, the trucks resultant displacement is 140 m east of origin

Preocupada com o aumento da tarifa na conta de luz, uma pessoa resolve economizar diminuindo o tempo de banho de 20 para 15 minutos. Seu chuveiro possui as seguintes especificações: 4200 W e 220V. Sabendo que o kWh custa R$0,30, a economia feita em 10 dias foi de aproximadamente​

Answers

Answer:

The mount saved is $ 0.105.

Explanation:

Concerned about the increase in the electricity bill, a person decides to save by reducing bathing time from 20 to 15 minutes. Your shower has the following specifications: 4200 W and 220V. Knowing that the kWh costs R$0.30, the savings made in 10 days were approximately​.

The electrical energy is given by

E = P x t

where, P is the electrical power and t is the time.

When he is using the shower for 20 minutes, the energy consumed is

E = 4200 x 20 x 60 = 5040,000 J = 1.4 kWh

When he is using the shower for 15 minutes, the energy consumed is

E' = 4200 x 15 x 60 = 3780000 J = 1.05 kWh

The difference in energy is

E'' = E - E' = 1.4 - 1.05 = 0.35 kWh

The money saved is

= 4 0.3 x 0.35 = $ 0.105

A loop of wire is in a magnetic field such that its axis is parallel with the field direction. Which of the following would result in an induced emf in the loop?
A. Moving the loop outside of the magnetic field region.
B. Change the diameter of the loop.
C. Change the magnitude of the magnetic field.
D. Spin the loop such that its axis does not consistently line up with the magnetic field direction.

Answers

Answer:

All the given options will result in an induced emf in the loop.

Explanation:

The induced emf in a conductor is directly proportional to the rate of change of flux.

[tex]emf = -\frac{d \phi}{dt} \\\\where;\\\\\phi \ is \ magnetic \ flux\\\\\phi = BA\ cos \theta[/tex]

where;

A is the area of the loop

B is the strength of the magnetic field

θ is the angle between the loop and the magnetic field

Considering option A, moving the loop outside the magnetic field will change the strength of the magnetic field and consequently result in an induced emf.

Considering option B, a change in diameter of the loop, will cause a change in the magnetic flux and in turn result in an induced emf.

Option C has a similar effect with option A, thus both will result in an induced emf.

Finally, considering option D, spinning the loop such that its axis does not consistently line up with the magnetic field direction will change the angle between the loop and the magnetic field. This effect will also result in an induced emf.

Therefore, all the given options will result in an induced emf in the loop.

In a certain cyclotron a proton moves in a circle of radius 0.530 m. The magnitude of the magnetic field is 1.30 T. (a) What is the oscillator frequency

Answers

Answer:

[tex]f=1.98\times 10^7\ Hz[/tex]

Explanation:

Given that,

The radius of circle, r = 0.53 m

The magnitude of the magnetic field, B = 1.3 T

We need to find the oscillator frequency. It is given by :

[tex]f=\dfrac{qB}{2\pi m}[/tex]

Put all the values,

[tex]f=\dfrac{1.6\times 10^{-19}\times 1.3}{2\pi \times 1.67\times 10^{-27}}\\\\f=1.98\times 10^7\ Hz[/tex]

So, the oscillator frequency is [tex]1.98\times 10^7\ Hz[/tex].

A piston-cylinder device contains 5 kg of refrigerant-134a at 0.7 MPa and 60°C. The refrigerant is now cooled at constant pressure until it exists as a liquid at 24°C. If the surroundings are at 100 kPa and-24°C, determine:
(a) the exergy of the refrigerant at the initial and the final states and
(b) the exergy destroyed during this process.

Answers

Answer:

Yes sure, keep it going, and never give up because your dreams are so important

A) The exergy of the refrigerant at the initial and final states are :

Initial state =  - 135.5285 kJ Final state =  -51.96 kJ

B) The exergy destroyed during this process is : - 1048.4397 kJ

Given data :

Mass ( M )  = 5 kg

P1 = 0.7 Mpa = P2

T1 = 60°C = 333 k

To = 24°C = 297 k

P2 = 100 kPa

A) Determine the exergy at initial and final states

At initial state :

U = 274.01 kJ/Kg , V = 0.034875 m³/kg , S = 1.0256 KJ/kg.k

exergy ( Ф ) at initial state = M ( U + P₂V - T₀S )

                                           = 5 ( 274.01 + 100* 10³ * 0.034875 - 297 * 1.0256)

                                           ≈ - 135.5285 kJ

At final state  :

U = 84.44 kJ / kg , V = 0.0008261 m³/kg,  S = 0.31958 kJ/kg.k

exergy ( ( Ф ) at final state = M ( U + P₂V - T₀S )

                                             = -51.96 kJ

B) Determine the exergy destroyed

  exergy destroyed = To * M ( S2 - S1 )

                                 = 297 * 5 ( 0.31958 - 1.0256 )

                                 = - 1048.4397 KJ

Hence we can conclude that A) The exergy of the refrigerant at the initial and final states are : Initial state =  - 135.5285 kJ, Final state =  -51.96 kJ  and The exergy destroyed during this process is : - 1048.4397 kJ

Learn more about exergy : https://brainly.com/question/25534266

Think about a thermos bottle. It consists of an inner bottle with a shiny silver surface separated from an outer container by a space with no air. In what ways does it block conduction, convection, and radiation?

Answers

Answer:

Radiation

Explanation:

Conduction, convection and radiation are the three modes of heat transfer.

1. Conduction: When the one end is heated of any rod, the heat transfer to the other end by the vibrational motion of the molecules, it is called conduction.

The heat transfer in a solid is due to the conduction.

2. Convection: When the liquid or gas is heated, the molecules which is in contact to the heat, heated first and due to the decrease in density they moves up and the molecules on the upper side are higher in density so they moves down. These are called convection currents. The process continues till the entire liquid becomes heated. It generally takes place in liquids and gases.  

3. Radiation: The process of heat transfer in which no molecules takes place is called radiation. The heat coming from sun is due to the radiation. It does not require any medium.  

In the thermos bottle, as there is no air between the two layers, so the heat transfer is due to the radiation.  

Part AFind the x- and y-components of the vector d⃗ = (4.0 km , 29 ∘ left of +y-axis).Express your answer using two significant figures. Enter the x and y components of the vector separated by a comma.d⃗ = km Part BFind the x- and y-components of the vector v⃗ = (2.0 cm/s , −x-direction).Express your answer using two significant figures. Enter the x and y components of the vector separated by a comma.v⃗ = cm/s Part CFind the x- and y-components of the vector a⃗ = (13 m/s2 , 36 ∘ left of −y-axis).Express your answer using two significant figures. Enter the x and y components of the vector separated by a comma.a⃗ x = m/s2

Answers

Solution :

Part A .

Given : The [tex]x[/tex] and [tex]y[/tex] components of the vector, d = [tex]\text{4 km 29}[/tex] degree left of [tex]y[/tex]-axis.

So the [tex]x[/tex] component is = -4 x sin (29°) = -1.939 km

           [tex]y[/tex] component is = 4 x cos (29°) = 3.498 km

Part B

Given : The [tex]x[/tex] and [tex]y[/tex] components of the vector, [tex]\text{v = 2 cm/s}[/tex] , [tex]\text{-x direction}[/tex]

So the [tex]x[/tex] component is = -2 cm/s

           [tex]y[/tex] component is = 0

Part C

Given : The [tex]x[/tex] and [tex]y[/tex] components of the vector, [tex]\text{a = 13 m/s, 36 degree}[/tex] left of [tex]y[/tex]-axis.

So the [tex]x[/tex] component is = -13 x sin (36°) = -7.6412 [tex]m/S^2[/tex]

           [tex]y[/tex] component is = -13 x cos (36°) = -10.517 [tex]m/S^2[/tex]

The x- and y-components of the vectors  is mathematically given as as follows for each Part respectively

x= -1.939 km, y= 3.498 km

x= -2 cm/s, 0

y=, x= -7.6412m/s^2, -10.517m/s^2

What are the x- and y-components of the vectors?

Question Parameters:

Generally, we follow a basic principle where

x component= Fsin\theta

y component= Fcos\theta

Therefore

For A

x component is

x= -4 x sin (29°)

x= -1.939 km

 y component is

y= 4 x cos (29°)

y= 3.498 km

For B

x component is

x= -2 cm/s            

y component is

y= 0

For C

x component is

x= -13 x sin (36°)

x= -7.6412m/s^2      

y component is

y= -13 x cos (36°)

y= -10.517m/s^2  

Read more about Cartession co ordinate

https://brainly.com/question/9410676

Other Questions
Midnight was as dark as a bottle of ink, compares _____. 9. During a titration, 50.0 ml of 0.2M NaOH were required to neutralize 50.0ml of H_{3}*P * O_{4} What's the concentration of the H_{3}*P * O_{4} solution? I need help with my math!!!!!!! I have a lot of algebra problems. Someone help me even with this one please! A plaintiff has brought an action for personal injuries against a store, in connection with an incident in which he slipped and fell after the store's linoleum floors had been mopped. A major issue at trial is the degree of moisture that remained on the floor, because it had been mopped 45 minutes before the plaintiff walked on it. The store offers the testimony of an expert, who will testify about an experiment he conducted measuring the amount of time necessary for a linoleum floor to dry completely after having been mopped. Under what condition should the court admit this testimony Jane is a full-time student on a somewhat limited budget, taking online classes, and she loves to play the latest video games. She needs to update her computer and wonders what to purchase. Which of the following might you suggest?a. A workstation with a 3.9 GHz quad-core processor, 16 gigabytes of RAM, and a 24" monitor. b. A laptop system with a 3.2 GHz processor. 6 gigabytes of RAM, and a 17" monitor. c. A desktop system with a 1.9 GHz processor, 1 gigabyte of RAM, and a 13" monitor d. A handheld PC (with no smartphone functionality) Do you think individual sports are more interesting than team sports to watch? Why?/Why not? ang sa mahigpit na sensur ssa mga akda o palabas How do you get 24 with the numbers 6, 5, 4 and 8 Can someone help me solve this problem please When extrication will involve cutting the roof off a vehicle, stabilization of the vehicle should include: Constitutional and absolute monarchies are similar because both haveO a democratic form of government that allows electorate controlO a ParliamentO monarchs with supreme unquestionable power over governmenta monarch Epithelial tissue is divided into which two categories?cardiacsimplesmoothstratified What is the wavelength of a wave that has a frequency of 6.1 x 109 Hz ? c = where c = 2.998 x 108 m/s a. 20.4 mb. 2.04 mc. 0.49 md. 0.049 m a total of 678 tickets were sold for the school play. They were either adult tickets or student tickets. there were 72 fewer student tickets sold than adult tickets. how many adult tickets were sold An English friend wants to learn your language. Write a letter detailing your experience of language learning, and explain what you think is the best way to go about it. Please can someone work this out please it would reallly help Assume there is a simultaneous decrease in the incomes of people in the market for new homes and a decrease in the wages paid to carpenters, plumbers, and electricians. All else constant, we can predict, with certainty, that in the market for new homes the equilibrium: in your own words, what is the definition for Double-helix? The reference desk of a university library receives requests for assistance. Assume that a Poisson probability distribution with an arrival rate of 10 requests per hour can be used to describe the arrival pattern and that service times follow an exponential probability distribution with a service rate of 12 requests per hour. What is the probability that no requests for assistance are in the system